7. At age 20, Heather began investing $3000 annually
into an account earning 7.5% interest compounded
annually. Lesley invested $6000 annually into a similar
account but began at age 40. They both stopped
contributing at age 65.
a) How much money did Heather and Lesley contribute
to their account?
b) What is the value of each of their investments when
they are 65 years old?
c) At age 65, when the investments mature, who has
more money and by how
much?

Answers

Answer 1

a) Heather contributed $135,000 and Lesley contributed $150,000 to their accounts.

b) Heather's investment is approximately $273,714.17, while Lesley's investment is approximately $191,048.18 when they are 65 years old.

c) Heather has more money by approximately $82,665.99 at age 65.

a) To find out how much money Heather and Lesley contributed to their accounts, we need to calculate the total contributions made by each of them.

Heather:

Heather started investing at age 20 and stopped at age 65, contributing $3000 annually. The number of years she contributed is (65 - 20) = 45 years.

Total contributions by Heather = $3000 × 45 = $135,000.

Lesley:

Lesley started investing at age 40 and stopped at age 65, contributing $6000 annually. The number of years she contributed is (65 - 40) = 25 years.

Total contributions by Lesley = $6000 × 25 = $150,000.

Therefore, Heather contributed $135,000 and Lesley contributed $150,000 to their respective accounts.

b) To calculate the value of their investments at age 65, we can use the formula for compound interest:

Future Value = Principal × (1 + interest rate)^number of years

Heather:

Principal (initial investment) = $3000

Interest rate = 7.5% = 0.075 (converted to decimal)

Number of years = 65 - 20 = 45

Future Value of Heather's investment = $3000 × (1 + 0.075)^45

Lesley:

Principal (initial investment) = $6000

Interest rate = 7.5% = 0.075 (converted to decimal)

Number of years = 65 - 40 = 25

Future Value of Lesley's investment = $6000 × (1 + 0.075)^25

Calculating these values:

Future Value of Heather's investment = $3000 × (1.075)^45 ≈ $273,714.17

Future Value of Lesley's investment = $6000 × (1.075)^25 ≈ $191,048.18

c) To determine who has more money at age 65 and by how much, we compare the future values of their investments.

Heather's investment value at age 65 = $273,714.17

Lesley's investment value at age 65 = $191,048.18

Therefore, Heather has more money at age 65, and the difference in their investments is approximately $273,714.17 - $191,048.18 = $82,665.99.

for such more question on Heather's investment

https://brainly.com/question/1400500

#SPJ8


Related Questions

Solve for each variable.
a = ___
b = ___
c = ___
d = ___

Answers

Answer:

a=55°

b=123°

c=55°

d=123°

El resultado de (6+9i) +(4-5i) es/
?

Answers

Answer:

es 362770

Step-by-step explanation:

saque esta imformacion de la calculadora

Answer:

10 + 4i

Step-by-step explanation:

To solve this, just combine the like terms:

[tex]\sf{(6+9i)+(4-5i)}[/tex]

[tex]\sf{6+9i+4-5i}[/tex]

[tex]\sf{6+4+9i-5i}[/tex]

[tex]\sf{10+4i}[/tex]

Hence, the answer is 10 + 4i

Sadie is going to see a movie and is taking her 4 kids. Each movie ticket costs $15 and there are an assortment of snacks available to purchase for $4.50 each. How much total money would Sadie have to pay for her family if she were to buy 6 snacks for everybody to share? How much would Sadie have to pay if she bought x x snacks for everybody to share?

Answers

Answer: 102 dollars/4.5x+75

Step-by-step explanation: Your question isn't really straightforward, x x snacks for everybody to share? Please elaborate, and are they talking about the total cost, or just the cost of 6 snacks?

First, we have to take into account that if SADIE is taking her FOUR kids, there will be 5 people.

Cost of tickets is equal to $15 per one, and

5x (where x=15, per 5)

5(15)=75, and now onto the snacks

4.5x2=9, and 6/2=3, so 9x3=27, or 4.5x6=27

75+27=102

then for x snacks, if 1 snack costs 4.5 dollars than it'd be 4.5x (x number of snacks)+75 to find the total cost with tickets and all.

What is the surface area of this cone?
Use 3.14 and round your answer to the nearest hundredth.
15 cm
9 cm

Answers

Answer:

answer is 749.07

just use the formula and put values for radius and height. you can find formula online or leave me a comment if you can't find it.

Jade decided to rent movies for a movie marathon over the weekend. The function g(x) represents the amount of money spent in dollars, where x is the number of movies. Does a possible solution of (6.5, $17.50) make sense for this function? Explain your answer.

Yes. The input and output are both feasible.
No. The input is not feasible.
No. The output is not feasible.
No. Neither the input nor output is feasible.

Answers

The output value is feasible. The input value is not feasible, the possible solution of (6.5, $17.50) does not make sense for this function. The correct answer is No. The input is not feasible.

Jade decided to rent movies for a movie marathon over the weekend.

The function g(x) represents the amount of money spent in dollars, where x is the number of movies.

The given function is g(x) which represents the amount of money spent in dollars, where x is the number of movies.

The solution given is (6.5, $17.50).

We need to find whether the solution makes sense for the given function or not.

The input is given as 6.5 and the output is given as $17.50.

This means that Jade rented 6.5 movies and spent $17.50 on renting those movies.

To check whether the solution makes sense or not, we need to see if the input and output values are feasible or not.

The input value 6.5 is not a feasible value because it is not possible to rent half a movie.

Jade can rent 6 movies or 7 movies but not 6.5 movies.

Therefore, the input value is not feasible.

On the other hand, the output value $17.50 is a feasible value because it is possible for Jade to spend $17.50 on renting 6 movies.

The output value is feasible.

Since the input value is not feasible, the possible solution of (6.5, $17.50) does not make sense for this function. The correct answer is No. The input is not feasible.

For more related questions on feasible:

https://brainly.com/question/32957392

#SPJ8

Answer the equation 2.5p + 1 = 1.4p

Answers

Answer:

p ≈ -0.9

Step-by-step explanation:

2.5p + 1 = 1.4p

2.5p = 1.4p - 1

1.1p = -1

p ≈ -0.9

So, p ≈ -0.9

Afish tank is 30 inches wide, 12 inches deep, and 18 inches tall Approximately how many gallons of water does it hold if there are 7 48 gallons per cubic foot of water?

39
28
19

Answers

Answer: 12

Step-by-step explanation:

12 is the answer 12…………..

the base of a square pyramid is 229 meters long, each slant height is 186 meters. what is the surface area​

Answers

Answer:

The total surface area is given by: base area + 4 * triangular face area

Substituting the values we calculated: 52441 + 4 * 10424.4 ≈ 91588.4 square meters.

Therefore, the surface area of the square pyramid is approximately 91588.4 square meters.

The total surface area is given by: base area +
4 * triangular face area
Substituting the values we calculated: 52441 +
4 * 10424.4 ~ 91588.4 square meters.
Therefore, the surface area of the square pyramid is approximately 91588.4 square meters.

Pablo solved the polynomial equations given in the table. Determine whether each polynomial is correct. Select Correct or Incorrect for each equation.

Equation

(b²+7b-9)+(4b-6b²) = -8b² + 14b-9

(4a+6)-(3a²-9a+1)=-3a²+ 13a +5

(12c-8c²)+(5c²- 10c) = -3c²+2c

Answers

The first equation is incorrect.

The second equation is correct.

The third equation is correct.

Let's go through each equation and determine whether it is correct or incorrect:

(b²+7b-9)+(4b-6b²) = -8b² + 14b-9

To determine if this equation is correct, we need to simplify both sides and check if they are equal.

On the left side:

(b²+7b-9)+(4b-6b²) = b² - 6b² + 7b + 4b - 9 = -5b² + 11b - 9

On the right side:

-8b² + 14b - 9

Comparing both sides, we can see that -5b² + 11b - 9 is not equal to -8b² + 14b - 9. Therefore, the equation is incorrect.

(4a+6)-(3a²-9a+1)=-3a²+ 13a +5

Again, we need to simplify both sides of the equation and check if they are equal.

On the left side:

(4a+6)-(3a²-9a+1) = 4a + 6 - 3a² + 9a - 1 = -3a² + 13a + 5

On the right side:

-3a² + 13a + 5

Comparing both sides, we can see that -3a² + 13a + 5 is equal to -3a² + 13a + 5. Therefore, the equation is correct.

(12c-8c²)+(5c²- 10c) = -3c²+2c

Again, let's simplify both sides and compare them.

On the left side:

(12c-8c²)+(5c²- 10c) = 12c - 8c² + 5c² - 10c = -3c² + 2c

On the right side:

-3c² + 2c

Comparing both sides, we can see that -3c² + 2c is equal to -3c² + 2c. Therefore, the equation is correct.

For more such questions on equation visit:

https://brainly.com/question/17145398

#SPJ8

Predict the population in 2016, as a decreases at a constant rate

Answers

Answer:

We need more information to accurately predict the population in 2016. The following information is needed:

The initial population (population in a given baseline year)

The known population decrease rate as a percentage or absolute number per year

For example, if:

The initial population (in 2010) was 10,000

The population is decreasing at a constant rate of 100 people per year

Then we can calculate the population in 2016 as follows:

2010 population: 10,000

2011 population: 10,000 - 100 = 9,900

2012 population: 9,900 - 100 = 9,800

2013 population: 9,800 - 100 = 9,700

2014 population: 9,700 - 100 = 9,600

2015 population: 9,600 - 100 = 9,500

2016 population: 9,500 - 100 = 9,400

Therefore, based on this information, the predicted population in 2016 would be 9,400.

In summary, to accurately predict population changes over time, we need to know the initial population and population decrease rate. With that information, we can calculate the population for each subsequent year by subtracting the decrease amount from the population in the previous year.

Hope this helps! Let me know if you have any other questions.

Step-by-step explanation:

Jennifer Aniston bought a property for $2,000,000. One year later, she sold it for $2,200,000. Jennifer invested only $1,000,000 of her own money and borrowed the rest interest-free from her friend, Brad Pitt. What was her return on this investment?

Answers

This means that she made a 20% return on the money she invested in the property. For every dollar she invested, she earned 20 cents in profit.

To calculate Jennifer Aniston's return on investment (ROI), we can use the formula:

ROI = (Net Profit / Initial Investment) * 100

First, let's calculate the net profit. The net profit is the selling price minus the initial investment:

Net Profit = Selling Price - Initial Investment

Net Profit = $2,200,000 - $2,000,000

Net Profit = $200,000

Next, we calculate the ROI:

ROI = (Net Profit / Initial Investment) * 100

ROI = ($200,000 / $1,000,000) * 100

ROI = 0.2 * 100

ROI = 20%

Jennifer Aniston's return on investment for this property is 20%.

For more such questions on invested visit:

https://brainly.com/question/25893158

#SPJ8

Here is Takeshi's work determining a third point on the graph of an exponential function, `h(x)`.



Explain why the work is incorrect.

Answers

Answer:

Step-by-step explanation:

Let h(x) = y

The exponentail function is of the form :

[tex]y = ab^x[/tex]

We have :

[tex]y_{_1} = ab^{x_{_1}}\\y_{_2} = ab^{x_{_2}}\\\\\implies \frac{y_{_1}}{y_{_2}} = \frac{ab^{x_{1}}}{ab^{x_{2}}} \\\\\implies \frac{y_{_1}}{y_{_2}} = \frac{b^{x_{1}}}{b^{x_{2}}} \\\\\implies \frac{y_{_1}}{y_{_2}} = b^{(x_1-x_2)}[/tex]

Given points : (4, 9) and (5, 34.2)

We have:

[tex]\frac{34.2}{9} = b^{(5-4)}\\\\\implies 3.8 = b[/tex]

Writing the equation with x, y and b:

[tex]y = ab^x\\\\\implies 9 = a(3.8^4)\\\\a = \frac{9}{3.8^4} \\\\a = 0.043[/tex]

a = 0.043

b = 3.8

When x = 6, y will be:

[tex]y = (0.043)(3.8^6)\\\\y = 128.47[/tex]

This is not the y value in the question y = 59.4

Therefore (6, 59.4) does not lie on the graph h(x)

Kemani Walker
Law of Sines
Jun 15, 9:29:00 PM
?
In ATUV, t = 820 inches, m/U=132° and m2V=25°. Find the length of u, to the
nearest inch.
Answer: u =
Submit Answer

Answers

The length of u, to the nearest inch, is 1818 inches.

To solve this problem, we can use the Law of Sines, which states that the ratio of the length of a side of a triangle to the sine of its opposite angle is constant.

In this case, we'll use the following formula:

a/sin(A) = b/sin(B) = c/sin(C)

Let's label the sides and angles of the triangle:

Side a = u (length of u)

Side b = t (820 inches)

Side c = v (length of v)

Angle A = m/U (132°)

Angle B = m2V (25°)

Angle C = 180° - A - B (as the sum of angles in a triangle is 180°)

Now, we can use the Law of Sines to set up the equation:

u/sin(A) = t/sin(B)

Plugging in the given values:

u/sin(132°) = 820/sin(25°)

To find the length of u, we'll solve this equation for u.

u = (820 [tex]\times[/tex] sin(132°)) / sin(25°)

Using a calculator, we can evaluate the right side of the equation to get the approximate value of u:

u ≈ (820 [tex]\times[/tex] 0.9397) / 0.4226

u ≈ 1817.54 inches

Rounding to the nearest inch, we have:

u ≈ 1818 inches

For similar question on Law of Sines.

https://brainly.com/question/30401249  

#SPJ8

D = {x|x is a whole number} E = {x|x is a perfect square between 1 and 9} F = {x|x is an even number greater than or equal to 2 and less than 9} Which of the following is an element of D ∩ (E ∩ F)? 16 3 6 4

Answers

The element 4 is an element of D ∩ (E ∩ F).

To find the intersection of sets D, E, and F, we need to determine the elements that are common to all three sets.

Set D consists of all whole numbers, so any whole number can be an element of set D.

Set E consists of perfect squares between 1 and 9. The perfect squares in this range are 1, 4, and 9.

Set F consists of even numbers greater than or equal to 2 and less than 9.

The even numbers in this range are 2, 4, 6, and 8.

Taking the intersection of sets E and F, we find that the common element is 4, as it is the only number that satisfies both conditions of being a perfect square and an even number in the given range.

Finally, taking the intersection of set D with the intersection of sets E and F, we find that the element 4 is also an element of set D ∩ (E ∩ F).

For similar question on element.

https://brainly.com/question/18096867  

#SPJ8

An experiment consists of rolling two fair number cubes. The diagram shows the sample space of all equally likely outcomes. What is the probability of rolling two 2's? Express your answer as a fraction in simplest form.

Answers

Since there are 6 possible outcomes for each roll of the number cube, there are 6 x 6 = 36 possible outcomes when two number cubes are rolled.

Out of these 36 outcomes, there is only one outcome where both cubes show 2. Therefore, the probability of rolling two 2's is:

1/36

So the answer is 1/36.

Need this soon!! AP Calc AB

Answers

Answer:

(Look at the picture)

Using a linear model, the predicted calories in food items based on grams of carbs are represented by the residual plot.
Residuals
30.00
20.00
10.00
0.00
-10.00
-20.00
-30.00
0
2
8
Carbs in grams
10
12
16
What does the pattern in the residual plot indicate about the type of model?

Answers

The pattern in the residual plot indicates the following about the type of model: C. The pattern is random, indicating a good fit for a linear model.

What is a residual plot?

In Mathematics and Geometry, a residual plot is sometimes referred to as scatter plot and it can be defined as a type of graph which is used for the graphical representation of the values of two (2) variables, with the resulting points showing any association or correlation (relationship) between the data set.

In Mathematics, a residual value is a difference between the measured (given or observed) value from a residual plot (scatter plot) and the predicted value from a residual plot (scatter plot).

By critically observing the residual plot which shows a relationship between the residuals and carbs in grams, we can logically deduce that a linear model is the most appropriate because there isn't a clear pattern between the data points i.e a random pattern.

Read more on residual plot here: https://brainly.com/question/16594090

#SPJ1

Complete Question:

What does the pattern in the residual plot indicate about the type of model?

A. The pattern is random, indicating a good fit for a nonlinear model.

B. The pattern shows the points are far from the zero line, indicating a good fit for a linear model.

C. The pattern is random, indicating a good fit for a linear model.

D. The pattern shows the points are far from the zero line, indicating a good fit for a nonlinear model.

E. The pattern is random, indicating that the model is unable to be determined.

Fill in the blank with the recursive function. Be sure to include the starting point.
Ashley has 100 books that she wants to give away at the rate of n books per week. Write a recursive function that represents the number of books Ashley has at any time.
The recursive function that gives the number of books Ashley has at any time is ____ = _____
, starting at ____ (I have this answer already, it is 100.)
100
.Every other response to this has been unsatisfactory, so I will elaborate further. This is on PLATOWEB.
If you're a student cramming and reading this for the answer, hi.

Answers

The recursive function that represents the number of books Ashley has at any time is Number of books(n) = Number of books(n-1) - n, starting at 100.

The recursive function that represents the number of books Ashley has at any time can be defined as follows:

Number of books(n) = Number of books(n-1) - n

Starting point: Number of books(0) = 100

Explanation:

In the recursive function, "n" represents the number of weeks that have passed. Each week, Ashley gives away "n" books. Therefore, the number of books she has at any time is equal to the number of books she had in the previous week (Number of books(n-1)) minus the number of books given away in the current week (n).

The starting point is given as Number of books(0) = 100, which means initially Ashley has 100 books before any weeks have passed.

for such more question on recursive function

https://brainly.com/question/25762866

#SPJ8

Write 0.000000624 in scientific notation

Answers

Answer:

6.24 x [tex]10^{-7}[/tex]

Step-by-step explanation:

0.000000624 in scientific notation is 6.24 x [tex]10^{-7}[/tex]

Answer:

Step-by-step explanation:

The resulting number is in scientific notation. In this case, the number is 6.24 * 10^-7.

What is the coefficient of x^2y^3 in the expression of (2x+y)^5

Answers

The coefficient of [tex]x^2y^3[/tex] in the expression [tex](2x+y)^5[/tex] is 10

To find the coefficient of the term containing [tex]x^2y^3\\[/tex] in the expansion of [tex](2x+y)^5[/tex], we can use the binomial theorem. The binomial theorem states that the expansion of [tex](a + b)^n[/tex] can be written as the sum of the binomial coefficients multiplied by the respective powers of a and b, where n is a positive integer.

In this case, we have [tex](2x + y)^5[/tex]. The term [tex]x^2y^3[/tex] can be obtained by selecting two x's from [tex](2x)^2[/tex] and three y's from [tex]y^3[/tex]. The coefficient of [tex]x^2y^3[/tex] will be the product of the corresponding binomial coefficients.

The binomial coefficients for this term can be calculated as follows:

Coefficient = C(5, 2) * C(3, 3)

C(n, k) represents the binomial coefficient, which is calculated as n! / (k! * (n - k)!), where "!" denotes the factorial operation.

Plugging in the values, we have:

Coefficient = C(5, 2) * C(3, 3)

[tex]= (5! / (2! * (5 - 2)!)) * (3! / (3! * (3 - 3)!))\\= (5! / (2! * 3!)) * (3! / (3! * 0!))\\= (5 * 4 * 3! / (2 * 1 * 3!)) * 1\\= (5 * 4) / (2 * 1)\\= 10.[/tex]

Therefore, the coefficient of [tex]x^2y^3[/tex] in the expression [tex](2x+y)^5[/tex] is 10.

To learn more about binomial theorem, refer:

brainly.com/question/222308

(4x³+6x²+20x+9)/2x+1
divide using long polynomial division

Answers

The result of dividing (4x³ + 6x² + 20x + 9) by (2x + 1) using long polynomial division is 2x² + 2x + 9 with a remainder of 0.

To divide the polynomial (4x³ + 6x² + 20x + 9) by (2x + 1) using long polynomial division.

Arrange the terms of the dividend and the divisor in descending order of the degree of x:

      2x + 1 | 4x³ + 6x² + 20x + 9

Divide the first term of the dividend by the first term of the divisor and write the result on the top line:

             2x + 1 | 4x³ + 6x² + 20x + 9

                   | 2x²

Multiply the divisor (2x + 1) by the quotient obtained in the previous step (2x²) and write the result below the dividend:

             2x + 1 | 4x³ + 6x² + 20x + 9

           - (4x³ + 2x²)

           ---------------

                        4x² + 20x + 9

Subtract the result obtained in the previous step from the dividend and bring down the next term.

             2x + 1 | 4x³ + 6x² + 20x + 9

           - (4x³ + 2x²)

           ---------------

                        4x² + 20x + 9

                      - (4x² + 2x)

                      ---------------

                               18x + 9

Repeat the process by dividing the term brought down (18x) by the first term of the divisor (2x):

             2x + 1 | 4x³ + 6x² + 20x + 9

           - (4x³ + 2x²)

           ---------------

                        4x² + 20x + 9

                      - (4x² + 2x)

                      ---------------

                               18x + 9

                             - (18x + 9)

                             ---------------

                                         0

The division is complete when the degree of the term brought down becomes less than the degree of the divisor.

In this case, the degree of the term brought down is 0 (a constant term). Since we can no longer divide further, the remainder is 0.

Therefore, the result of the division is:

Quotient: 2x² + 2x + 9

Remainder: 0

For similar question on polynomial division.

https://brainly.com/question/24662212  

#SPJ8

Help, please !!!!
A scatter plot is shown on the coordinate plane.

scatter plot with points at 1 comma 9, 2 comma 7, 3 comma 5, 3 comma 9, 4 comma 3, 5 comma 7, 6 comma 5, and 9 comma 5

Which two points would a line of fit go through to best fit the data?

(1, 9) and (9, 5)
(1, 9) and (5, 7)
(2, 7) and (4, 3)
(2, 7) and (6, 5)

Answers

Answer:

(2,7) and (6,5)

Step-by-step explanation:

The line of best fit would be approximately:

y = -.4x + 8

(1,9)

9 = -.4(1) + 8

9 = 7.6

(9,5)

y = -.4x + 8

5 = -.4(9) + 8

5 = 4.4

(5,7)

y = -.4x + 8

7 = -.4(5) + 8

7 = 6

(2,7)

y = -.4x + 8

7 = -.4(2) + 8

7 = 7.2

(4,3)

y = -.4x + 8

3 = -.4(4) + 8

3 = 6.4

(6,5)

y = -.4x + 8

5 = -.4(6) + 8

5 = 5.6

How to represent 7/3 as a decimal

Answers

Fraction 7/3 is convertible to a decimal. It is a proper fraction. Because here the numerator is lesser than the denominator. Any fraction can be represented as a decimal number.

To transform the fraction into a decimal, it has to go through a process of division. It will be like this, 7÷3.

During the process of the division after the use of the decimal point, the same number will keep repeating. The number that keeps repeating is known as the recurring number.

If 7 is divided by 3, the result is 2.333333. Here the digit 3 is recurring. So the representation of 7/3 as a decimal is 2.333333.

Try more decimal sums:

https://brainly.com/question/28393353

Refer to the table summarizing service times​ (seconds) of dinners at a fast food restaurant. How many individuals are included in the​ summary? Is it possible to identify the exact values of all of the original service​ times?
Time (sec) Frequency
60 to 119 7
120 to 179 24
180 to 239 14
240 to 299 1
300 to 359 4

Answers

Answer:

Based on the provided information, the table summarizes service times (in seconds) of dinners at a fast food restaurant. To determine the number of individuals included in the summary, we can sum up the frequencies listed in the table:

7 + 24 + 14 + 1 + 4 = 50

Therefore, there are 50 individuals included in the summary.

Regarding the exact values of all the original service times, it is not possible to determine them precisely based on the given information. The table only provides ranges of service times and their corresponding frequencies. We can determine the range within which each individual's service time falls, but we cannot determine the exact value within that range.

Which of the following is the prime factorization of 15?
01x15
03x5
02x2x5
05x2

Answers

Answer: The second one, 3×5

Step-by-step explanation:

We can use a factor tree to determine the prime factorization of 15. We first choose factors of 15, and the only factors we can use besides 1 and 15 are 3 and 5.

                                                             15

                                                            3×5

That is all, because the final numbers listed are prime and we cannot perform any further actions. The prime factorization of 15 is 3×5, or in exponential form, [tex]3^1[/tex]×[tex]5^1[/tex].

Note: You may notice with other numbers that there are several factors to choose from. That's okay, because in the end it will give you the same result.

Hope this helps!

Is there a difference between shapes when plotting Uniform acceleration towards (+)directtion,Uniform acceleration towards (-)direction, Uniform deceleration towards (+) direction and Uniform deceleration towards (-) direction in displacement time graph

Answers

Yes, there is a difference in the shapes of the displacement-time graphs for uniform acceleration towards the positive direction, uniform acceleration towards the negative direction, uniform deceleration towards the positive direction, and uniform deceleration towards the negative direction.

Uniform acceleration towards the positive direction:

In this case, the object's velocity increases in the positive direction over time. The displacement-time graph will have a concave-upward shape, forming a curve that starts with a small slope and gradually becomes steeper as time progresses.

Uniform acceleration towards the negative direction:

Here, the object's velocity increases in the negative direction, meaning it accelerates in the opposite direction to its positive direction.

The displacement-time graph will have a concave-downward shape, forming a curve that starts with a steep slope and gradually becomes less steep as time progresses.

Uniform deceleration towards the positive direction:

In this scenario, the object's velocity decreases in the positive direction, but it still moves towards the positive direction.

The displacement-time graph will show a curve with a decreasing slope, forming a concave-downward shape, indicating that the object is slowing down.

Uniform deceleration towards the negative direction:

Here, the object's velocity decreases in the negative direction, opposing its initial direction.

The displacement-time graph will have a curve with a decreasing slope, forming a concave-upward shape, indicating that the object is slowing down but still moving in the negative direction.

In summary, the shapes of the displacement-time graphs differ based on the direction and type of acceleration (positive or negative) and whether the object is undergoing uniform acceleration or uniform deceleration. These differences can be observed through the concavity and slope of the graphs.

For similar question on displacement-time.

https://brainly.com/question/28357012  

#SPJ8

please explain the logic behind it

Answers

first of all, you need to assume that this shape is like a cone. So cone has a height and you can find it by using Thales's theorem. so it would be

[tex] \frac{x}{x + 6} = \frac{4}{8} [/tex]

x is 6. So the total height of the cone has founded.

To find the volume of this frustum we have to subtract the volume of the small cone(at the top of the frustum) which has 6 cm in height from the big cone with 12 cm in height.

big cone Volume: ⅓(8²π×12)

-

small cone Volume: ⅓(4²π × 6)

=

frustum volume : 224π

Washington, DC is 389 miles from Statesville, NC. If you wanted to drive there,
how long would it take you driving on interstates with an average of 65 mph?
O 5.98 hours
07.07 hours
O 5.56 hours
07.78 hours
O 6.48 hours
Suppose your car gets 29 miles per gallon on the interstate and gas costs
$3.89/gallon. How much will it cost you to drive to Washington, DC?
O $0.29
O $43,883.09
$52.18
O $3.45
O $2,900.00

Answers

It would cost approximately $52.18 to drive from Statesville, NC to Washington, DC with a car that gets 29 miles per gallon on the interstate, considering a gas cost of $3.89 per gallon.

To calculate the time it would take to drive from Statesville, NC to Washington, DC, we can divide the distance of 389 miles by the average speed of 65 mph.

Time = Distance / Speed

Time = 389 miles / 65 mph

Time ≈ 5.98 hours

Therefore, it would take approximately 5.98 hours to drive from Statesville, NC to Washington, DC on interstates with an average speed of 65 mph.

To calculate the cost of driving to Washington, DC, we need to know the number of gallons of gas required for the trip. We can find this by dividing the distance by the car's mileage, which is 29 miles per gallon.

Number of gallons = Distance / Mileage

Number of gallons = 389 miles / 29 miles per gallon

Number of gallons ≈ 13.41 gallons

The cost of gas can be calculated by multiplying the number of gallons by the cost per gallon, which is $3.89.

Cost = Number of gallons * Cost per gallon

Cost ≈ 13.41 gallons * $3.89/gallon

Cost ≈ $52.18

Therefore, it would cost approximately $52.18 to drive from Statesville, NC to Washington, DC with a car that gets 29 miles per gallon on the interstate, considering a gas cost of $3.89 per gallon.

for such more question on cost

https://brainly.com/question/25109150

#SPJ8

What number after being increased by 22% results in a value of 305?

Answers

Answer:

250

Step-by-step explanation:

[tex]x+0.22x=305\\1.22x=305\\x=250[/tex]

Answer:

250

Step-by-step explanation:

so when you add 22% to 250 it equals 305

really need help with is problem if any math wizards are on

Answers

[tex]f(4)=4\\f(-5)=1\\x=-4,2[/tex]

Other Questions
The purpose of a Volutrol (Buretrol) administration set is to:A)avoid inadvertent fluid overload.B)deliver large volumes of IV fluid.C)facilitate piggyback medication infusions.D)deliver a maximum of 50 mL of IV solution g determine a molecular formula, e.g. ch4, from the line structure below. list elements in the order ch(others in alphabetical order). use the subscript button in the box below to format your answer properly. what happened when hungry opened its borders n which the class should have a constructor that accepts a non-negative integer and uses it to initialize the Numbers object's data member. It also should have a member function print() that prints the English description of the number instance variable number. Demonstrate the class by writing a main program that asks the user to enter a number in the proper range and then prints its English description on the console. The program should then loop back and prompt for the next number to translate. The programs stop when the user enters a 0 (zero) -- this is called the "sentinel". Submit a screen snip using the following test cases: 5, 10, 11, 18, 60, 295, 970, 1413, 6000, 9999, and 0.#include using namespace std;class Numbers //Creating class{public :int number;static string lessThan20[20] ;static string tens[10] ;static string hundred ;static string thousand;Numbers(int a) //Parameterized constructor for initializing value of number{number = a;}void print(){string str = "";if(number>999) //thousands place{str = str +" "+ lessThan20[number/1000] +" "+ thousand+" ";number=number%1000;}if(number>99)//hundreds place{str = str +" "+ lessThan20[number/100] + " "+hundred+" ";number=number%100;}if (number0) //if number is zero we do not want it to add word zero examppke for 80 we dont want eighty zero as outputstr+= lessThan20[number%10] ;}cout<}};//setting values for static variablesstring Numbers::lessThan20[] = {"zero","one","two","three","four","five","six","seven","eight","nine","ten","eleven","twelve","thirteen","fourteen","fifteen","sixteen","seventeen","eighteen","nineteen"};string Numbers::tens[] = { "","","twenty ", "thirty ", "forty ", "fifty ", "sixty ", "seventy ", "eighty ", "ninety " };string Numbers::thousand = "thousand";string Numbers::hundred = "hundred";int main(){int n;coutn; //take input from userNumbers obj1(n); //create object of classobj1.print();} For this discussion post, we are going to run a hypothesis test based on a claim made by an insurance provider. Read the following:An insurance provider states that their customers save at least, on average, 300 dollars per year by switching to them, with a standard deviation of 150 dollars. Before we decide to switch to the new company and go through all of the hassle, we want to test the claim. So, we go out and sample 64 individuals who switched to the new insurance company and found them to have saved an average of 255 dollars per year. Do we have enough evidence at the = 0. 05 level to state that the insurance provider is false in their claim?Discussion PromptsAnswer the following questions in your initial post:1. What are the hypotheses based on the words given in the problem?2. Should we use a Z or T distribution in this case?3. What is our Z or T statistic?4. What is the P-value?5. Based on your p-value and alpha, what conclusion will we make?6. Based on your results, would you switch to this company? Explain why or why not (Note: this can go beyond the use of statistics, but statistical analysis can help our decisions) What is the output of this program? (fill the box on right). 2. Write a recurrence [equation] for the function bar(n). 3. What is the type (name) of this recurrence? list six aids to trade A symmetrical image, with shapes emerging from a center, used as a meditational focuso Visual aids to concentration and illumination, which portray Buddhas and bodhisattvas and representations of an ideal universeo Used by monks and lay peopleo Mahayana Buddhism [TibetVajrayana Buddhism=varient of Mahayana] A drainage basin is defined bya) its proximity to a coastb) the salinity of the stream waterc) the steepness of the stream bedd) its drainage divides Distinguish between the Active style and Passive style Investing and provide your insight on which approach is appropriate considering different Investor profiles with different risk appetites. Use relevant examples to demonstrate your understanding of different strategies under active and passive investing. Consider the following SystemVerilog modules:module bottom(input logic a,output logic y);assign y = ~a;endmodulemodule top();logic y,a;assign y = 1'b1;bottom mything (.a(y),.y(a));endmoduleWhat is the value of "a" and "y" within "mything"?a.0 and 1b.X and Xc.1 and 1d.1 and 0e.Z and Zf.0 and 0 The following information was extracted from the books of KASTA Business on 31 December Additional information: (i) Withdrawal of RM5,000 from bank account has not been recorded. (ii) Advertising expenses RM65,000 were outstanding. (iii) All non-current assets are depreciated at 30% per annum, using reducing balance method. (v) Closing Inventory RM84,300 fully counted on 31 December. Required: Show all your working. (a) Prepare the Income Statement for the year ended 31 December 2021, and (b) the Balance Sheet as at 31 December 2021 . cyclohexanol sulfuric acid cyclohexene molar mass: molar mass: molar mass: volume: volume: density: density: moles: boiling point: concentration: moles: boiling point: boiling point: Pottery Corporation sells many of its pottery overseas. The following transaction took placeOn April 13, Pottery sold pottery planters to a company in Spain for 40,000 with payment due on June 15.Also on April 13, Pottery entered into a 60 day forward contract to sell 40,000 at a forward rate of1 = 1.350. The forward contract is not designated as a hedge. The spot rates follow:April 13 1 = 1.343June 15 1 = 1.348Required: Journal entries for the transactions. Earlier in the semester, we construct a vector that is parallel to the line of intersections of two planes. Now, construct a vector that is parallel to the curve of intersection of the surfaces z = x + y and z=2xy +1 at (1,1,2). Calculate the volume of the solid ER3 bounded by the planes y=0, z=0, z=1xy and the parabolic cylinder y = 1 x^2. this traditional power allows the president to withhold certain information. what is the opposite of instant Solve the system below xy+z=03x+y+2z=22x+yz=3 For self interest threat, self-review threat, self evaluation threat, familiarity threat and intimidation threat , suggest a safeguard that can be put in place to combat the threat identified.